Sequence such that every subsequence can have a different real limit [duplicate]

The name of the pictureThe name of the pictureThe name of the pictureClash Royale CLAN TAG#URR8PPP











up vote
8
down vote

favorite
1













This question already has an answer here:



  • Give an example of a sequence of real numbers with subsequences converging to every real number

    3 answers



I would like to find a sequence of real numbers $(a_n)_ninmathbbN$ with this property: for any $LinmathbbR$ there is a subsequence $a_k_n$ such that $$lim_ntoinfty a_k_n = L$$ Does such a sequence exist?










share|cite|improve this question















marked as duplicate by Martin R, Community Dec 9 at 14:29


This question has been asked before and already has an answer. If those answers do not fully address your question, please ask a new question.














  • Otherwise said, the set $ ninmathbb N$ should be dense in $mathbb R$.
    – Giuseppe Negro
    Dec 9 at 11:49














up vote
8
down vote

favorite
1













This question already has an answer here:



  • Give an example of a sequence of real numbers with subsequences converging to every real number

    3 answers



I would like to find a sequence of real numbers $(a_n)_ninmathbbN$ with this property: for any $LinmathbbR$ there is a subsequence $a_k_n$ such that $$lim_ntoinfty a_k_n = L$$ Does such a sequence exist?










share|cite|improve this question















marked as duplicate by Martin R, Community Dec 9 at 14:29


This question has been asked before and already has an answer. If those answers do not fully address your question, please ask a new question.














  • Otherwise said, the set $ ninmathbb N$ should be dense in $mathbb R$.
    – Giuseppe Negro
    Dec 9 at 11:49












up vote
8
down vote

favorite
1









up vote
8
down vote

favorite
1






1






This question already has an answer here:



  • Give an example of a sequence of real numbers with subsequences converging to every real number

    3 answers



I would like to find a sequence of real numbers $(a_n)_ninmathbbN$ with this property: for any $LinmathbbR$ there is a subsequence $a_k_n$ such that $$lim_ntoinfty a_k_n = L$$ Does such a sequence exist?










share|cite|improve this question
















This question already has an answer here:



  • Give an example of a sequence of real numbers with subsequences converging to every real number

    3 answers



I would like to find a sequence of real numbers $(a_n)_ninmathbbN$ with this property: for any $LinmathbbR$ there is a subsequence $a_k_n$ such that $$lim_ntoinfty a_k_n = L$$ Does such a sequence exist?





This question already has an answer here:



  • Give an example of a sequence of real numbers with subsequences converging to every real number

    3 answers







sequences-and-series limits analysis






share|cite|improve this question















share|cite|improve this question













share|cite|improve this question




share|cite|improve this question








edited Dec 9 at 11:46









Especially Lime

21.3k22656




21.3k22656










asked Dec 9 at 11:31









Riccardo Cazzin

1905




1905




marked as duplicate by Martin R, Community Dec 9 at 14:29


This question has been asked before and already has an answer. If those answers do not fully address your question, please ask a new question.






marked as duplicate by Martin R, Community Dec 9 at 14:29


This question has been asked before and already has an answer. If those answers do not fully address your question, please ask a new question.













  • Otherwise said, the set $ ninmathbb N$ should be dense in $mathbb R$.
    – Giuseppe Negro
    Dec 9 at 11:49
















  • Otherwise said, the set $ ninmathbb N$ should be dense in $mathbb R$.
    – Giuseppe Negro
    Dec 9 at 11:49















Otherwise said, the set $ ninmathbb N$ should be dense in $mathbb R$.
– Giuseppe Negro
Dec 9 at 11:49




Otherwise said, the set $ ninmathbb N$ should be dense in $mathbb R$.
– Giuseppe Negro
Dec 9 at 11:49










4 Answers
4






active

oldest

votes

















up vote
9
down vote



accepted










Just arrange the set of rational numbers in a sequence $a_n$. Given any real number $L$ and any positive integer $n$ there are infinitely many rationals in $(L-frac 1 n, L+frac 1 n)$. Pick $a_n_1$ in this interval with $n=1$. Then consider the case $n=2$. You can surely find $n_2 >n_1$ such that $a_n_2 in (L-frac 1 2, L+frac 1 2)$. Use induction to construct a subsequence $a_n_k$ such that $|a_n_k-L| <frac 1 k$ for all $k$. Then $a_n_k to L$.






share|cite|improve this answer






















  • Could you please explain better how it would work? Thanks
    – gimusi
    Dec 9 at 11:40










  • That's nice, we are then using that rationals are countable. Thanks for the clarification. Regards
    – gimusi
    Dec 9 at 12:00










  • Most welcome, @gimusi !
    – Kavi Rama Murthy
    Dec 9 at 12:04

















up vote
4
down vote













Take the sequence that sweep the interval $[-1,1]$ by $1/2$ steps, then the interval $[-2,2]$ by $1/2^2$ steps, then the interval $[-n,n]$ by $1/2^n$ steps... and so on.



You’ll be able to prove that every real is a limit point of that sequence.






share|cite|improve this answer




















  • That's less intuitive at first!
    – gimusi
    Dec 9 at 12:01

















up vote
3
down vote













Rephrasing:



Consider $a_n$, $n=1,2,3,3,....,$ the sequence of rational numbers. Recall that $mathbbQ$ is countable, hence can be written as a sequence $a_n$, $nin mathbbN$.



Let $L in mathbbR$.



Since $mathbbQ$ is dense in $mathbbR$, we can construct a subsequence $a_n_k$ that converges to $L$.






share|cite|improve this answer



























    up vote
    1
    down vote













    Consider a general topological space $(X, mathscrT)$ and a sequence of points $x in X^mathbbN$. One says that point $t in X$ is adherent to the sequence $x$ (some authors use the terminology 'cluster-point', but I don't fancy it so much) if:



    $$(forall V, n)(V in mathscrV_mathscrT(x) wedge n in mathbbN implies (exists m)(m geqslant n wedge x_m in V ))$$



    where $mathscrV_mathscrT(x)$ symbolizes the filter of neighbourhoods of point $x$ induced by the topology $mathscrT$. In a more descriptive fashion, $t$ is adherent to sequence $x$ if any neighbourhood of $t$ contains terms of arbitrarily high rank from the sequence $x$. If the filter of neighbourhoods of $t$ admits a countable base, then $t$ can be expressed as the limit of a subsequence of $x$. Therefore, in a space satisfying the First Axiom of Countability (i.e. all points have a countable base of neighbourhoods), the points adherent to a given sequence $x$ can be equivalently characterised as limits of subsequences of $x$.



    Now, if the space $X$ is non-empty and separable, let us fix a certain dense subset $T subseteq X$. As $X$ is non-empty, so must $T$ be. It is not difficult to show that a non-empty countable set $M$ admits a surjection $sigma: mathbbN rightarrow M$ such that for each $t in M$ the fibre $sigma^-1(t)$ be infinite.



    Consider such a surjection $sigma: mathbbN rightarrow T$ and define the sequence $t=(sigma(n))_n in mathbbN$ (which is actually the graphic of map $sigma$). The condition on the cardinality of the fibers ensures that any element $x in T$ is the limit of a (constant) subsequence of $t$. If we furthermore assume that the space $(X, mathscrT)$ is $T_1$, then any $x in X setminus T$ will be an accumulation point of $T$ and thus necessarily adherent to sequence $t$.



    To conclude, given a topological space $(X, mathscrT)$ that is non-empty, separable, first countable and $T_1$, one can always find a sequence $t$ of points within the space such that each element $x in X$ be expressible as the limit of a subsequence of $t$. This applies in particular to the topological space $(mathbbR, mathscrO)$, where $mathscrO$ denotes the (usual) order topology.






    share|cite|improve this answer



























      4 Answers
      4






      active

      oldest

      votes








      4 Answers
      4






      active

      oldest

      votes









      active

      oldest

      votes






      active

      oldest

      votes








      up vote
      9
      down vote



      accepted










      Just arrange the set of rational numbers in a sequence $a_n$. Given any real number $L$ and any positive integer $n$ there are infinitely many rationals in $(L-frac 1 n, L+frac 1 n)$. Pick $a_n_1$ in this interval with $n=1$. Then consider the case $n=2$. You can surely find $n_2 >n_1$ such that $a_n_2 in (L-frac 1 2, L+frac 1 2)$. Use induction to construct a subsequence $a_n_k$ such that $|a_n_k-L| <frac 1 k$ for all $k$. Then $a_n_k to L$.






      share|cite|improve this answer






















      • Could you please explain better how it would work? Thanks
        – gimusi
        Dec 9 at 11:40










      • That's nice, we are then using that rationals are countable. Thanks for the clarification. Regards
        – gimusi
        Dec 9 at 12:00










      • Most welcome, @gimusi !
        – Kavi Rama Murthy
        Dec 9 at 12:04














      up vote
      9
      down vote



      accepted










      Just arrange the set of rational numbers in a sequence $a_n$. Given any real number $L$ and any positive integer $n$ there are infinitely many rationals in $(L-frac 1 n, L+frac 1 n)$. Pick $a_n_1$ in this interval with $n=1$. Then consider the case $n=2$. You can surely find $n_2 >n_1$ such that $a_n_2 in (L-frac 1 2, L+frac 1 2)$. Use induction to construct a subsequence $a_n_k$ such that $|a_n_k-L| <frac 1 k$ for all $k$. Then $a_n_k to L$.






      share|cite|improve this answer






















      • Could you please explain better how it would work? Thanks
        – gimusi
        Dec 9 at 11:40










      • That's nice, we are then using that rationals are countable. Thanks for the clarification. Regards
        – gimusi
        Dec 9 at 12:00










      • Most welcome, @gimusi !
        – Kavi Rama Murthy
        Dec 9 at 12:04












      up vote
      9
      down vote



      accepted







      up vote
      9
      down vote



      accepted






      Just arrange the set of rational numbers in a sequence $a_n$. Given any real number $L$ and any positive integer $n$ there are infinitely many rationals in $(L-frac 1 n, L+frac 1 n)$. Pick $a_n_1$ in this interval with $n=1$. Then consider the case $n=2$. You can surely find $n_2 >n_1$ such that $a_n_2 in (L-frac 1 2, L+frac 1 2)$. Use induction to construct a subsequence $a_n_k$ such that $|a_n_k-L| <frac 1 k$ for all $k$. Then $a_n_k to L$.






      share|cite|improve this answer














      Just arrange the set of rational numbers in a sequence $a_n$. Given any real number $L$ and any positive integer $n$ there are infinitely many rationals in $(L-frac 1 n, L+frac 1 n)$. Pick $a_n_1$ in this interval with $n=1$. Then consider the case $n=2$. You can surely find $n_2 >n_1$ such that $a_n_2 in (L-frac 1 2, L+frac 1 2)$. Use induction to construct a subsequence $a_n_k$ such that $|a_n_k-L| <frac 1 k$ for all $k$. Then $a_n_k to L$.







      share|cite|improve this answer














      share|cite|improve this answer



      share|cite|improve this answer








      edited Dec 9 at 11:45

























      answered Dec 9 at 11:37









      Kavi Rama Murthy

      47.8k31854




      47.8k31854











      • Could you please explain better how it would work? Thanks
        – gimusi
        Dec 9 at 11:40










      • That's nice, we are then using that rationals are countable. Thanks for the clarification. Regards
        – gimusi
        Dec 9 at 12:00










      • Most welcome, @gimusi !
        – Kavi Rama Murthy
        Dec 9 at 12:04
















      • Could you please explain better how it would work? Thanks
        – gimusi
        Dec 9 at 11:40










      • That's nice, we are then using that rationals are countable. Thanks for the clarification. Regards
        – gimusi
        Dec 9 at 12:00










      • Most welcome, @gimusi !
        – Kavi Rama Murthy
        Dec 9 at 12:04















      Could you please explain better how it would work? Thanks
      – gimusi
      Dec 9 at 11:40




      Could you please explain better how it would work? Thanks
      – gimusi
      Dec 9 at 11:40












      That's nice, we are then using that rationals are countable. Thanks for the clarification. Regards
      – gimusi
      Dec 9 at 12:00




      That's nice, we are then using that rationals are countable. Thanks for the clarification. Regards
      – gimusi
      Dec 9 at 12:00












      Most welcome, @gimusi !
      – Kavi Rama Murthy
      Dec 9 at 12:04




      Most welcome, @gimusi !
      – Kavi Rama Murthy
      Dec 9 at 12:04










      up vote
      4
      down vote













      Take the sequence that sweep the interval $[-1,1]$ by $1/2$ steps, then the interval $[-2,2]$ by $1/2^2$ steps, then the interval $[-n,n]$ by $1/2^n$ steps... and so on.



      You’ll be able to prove that every real is a limit point of that sequence.






      share|cite|improve this answer




















      • That's less intuitive at first!
        – gimusi
        Dec 9 at 12:01














      up vote
      4
      down vote













      Take the sequence that sweep the interval $[-1,1]$ by $1/2$ steps, then the interval $[-2,2]$ by $1/2^2$ steps, then the interval $[-n,n]$ by $1/2^n$ steps... and so on.



      You’ll be able to prove that every real is a limit point of that sequence.






      share|cite|improve this answer




















      • That's less intuitive at first!
        – gimusi
        Dec 9 at 12:01












      up vote
      4
      down vote










      up vote
      4
      down vote









      Take the sequence that sweep the interval $[-1,1]$ by $1/2$ steps, then the interval $[-2,2]$ by $1/2^2$ steps, then the interval $[-n,n]$ by $1/2^n$ steps... and so on.



      You’ll be able to prove that every real is a limit point of that sequence.






      share|cite|improve this answer












      Take the sequence that sweep the interval $[-1,1]$ by $1/2$ steps, then the interval $[-2,2]$ by $1/2^2$ steps, then the interval $[-n,n]$ by $1/2^n$ steps... and so on.



      You’ll be able to prove that every real is a limit point of that sequence.







      share|cite|improve this answer












      share|cite|improve this answer



      share|cite|improve this answer










      answered Dec 9 at 11:42









      mathcounterexamples.net

      23.9k21753




      23.9k21753











      • That's less intuitive at first!
        – gimusi
        Dec 9 at 12:01
















      • That's less intuitive at first!
        – gimusi
        Dec 9 at 12:01















      That's less intuitive at first!
      – gimusi
      Dec 9 at 12:01




      That's less intuitive at first!
      – gimusi
      Dec 9 at 12:01










      up vote
      3
      down vote













      Rephrasing:



      Consider $a_n$, $n=1,2,3,3,....,$ the sequence of rational numbers. Recall that $mathbbQ$ is countable, hence can be written as a sequence $a_n$, $nin mathbbN$.



      Let $L in mathbbR$.



      Since $mathbbQ$ is dense in $mathbbR$, we can construct a subsequence $a_n_k$ that converges to $L$.






      share|cite|improve this answer
























        up vote
        3
        down vote













        Rephrasing:



        Consider $a_n$, $n=1,2,3,3,....,$ the sequence of rational numbers. Recall that $mathbbQ$ is countable, hence can be written as a sequence $a_n$, $nin mathbbN$.



        Let $L in mathbbR$.



        Since $mathbbQ$ is dense in $mathbbR$, we can construct a subsequence $a_n_k$ that converges to $L$.






        share|cite|improve this answer






















          up vote
          3
          down vote










          up vote
          3
          down vote









          Rephrasing:



          Consider $a_n$, $n=1,2,3,3,....,$ the sequence of rational numbers. Recall that $mathbbQ$ is countable, hence can be written as a sequence $a_n$, $nin mathbbN$.



          Let $L in mathbbR$.



          Since $mathbbQ$ is dense in $mathbbR$, we can construct a subsequence $a_n_k$ that converges to $L$.






          share|cite|improve this answer












          Rephrasing:



          Consider $a_n$, $n=1,2,3,3,....,$ the sequence of rational numbers. Recall that $mathbbQ$ is countable, hence can be written as a sequence $a_n$, $nin mathbbN$.



          Let $L in mathbbR$.



          Since $mathbbQ$ is dense in $mathbbR$, we can construct a subsequence $a_n_k$ that converges to $L$.







          share|cite|improve this answer












          share|cite|improve this answer



          share|cite|improve this answer










          answered Dec 9 at 11:51









          Peter Szilas

          10.6k2720




          10.6k2720




















              up vote
              1
              down vote













              Consider a general topological space $(X, mathscrT)$ and a sequence of points $x in X^mathbbN$. One says that point $t in X$ is adherent to the sequence $x$ (some authors use the terminology 'cluster-point', but I don't fancy it so much) if:



              $$(forall V, n)(V in mathscrV_mathscrT(x) wedge n in mathbbN implies (exists m)(m geqslant n wedge x_m in V ))$$



              where $mathscrV_mathscrT(x)$ symbolizes the filter of neighbourhoods of point $x$ induced by the topology $mathscrT$. In a more descriptive fashion, $t$ is adherent to sequence $x$ if any neighbourhood of $t$ contains terms of arbitrarily high rank from the sequence $x$. If the filter of neighbourhoods of $t$ admits a countable base, then $t$ can be expressed as the limit of a subsequence of $x$. Therefore, in a space satisfying the First Axiom of Countability (i.e. all points have a countable base of neighbourhoods), the points adherent to a given sequence $x$ can be equivalently characterised as limits of subsequences of $x$.



              Now, if the space $X$ is non-empty and separable, let us fix a certain dense subset $T subseteq X$. As $X$ is non-empty, so must $T$ be. It is not difficult to show that a non-empty countable set $M$ admits a surjection $sigma: mathbbN rightarrow M$ such that for each $t in M$ the fibre $sigma^-1(t)$ be infinite.



              Consider such a surjection $sigma: mathbbN rightarrow T$ and define the sequence $t=(sigma(n))_n in mathbbN$ (which is actually the graphic of map $sigma$). The condition on the cardinality of the fibers ensures that any element $x in T$ is the limit of a (constant) subsequence of $t$. If we furthermore assume that the space $(X, mathscrT)$ is $T_1$, then any $x in X setminus T$ will be an accumulation point of $T$ and thus necessarily adherent to sequence $t$.



              To conclude, given a topological space $(X, mathscrT)$ that is non-empty, separable, first countable and $T_1$, one can always find a sequence $t$ of points within the space such that each element $x in X$ be expressible as the limit of a subsequence of $t$. This applies in particular to the topological space $(mathbbR, mathscrO)$, where $mathscrO$ denotes the (usual) order topology.






              share|cite|improve this answer
























                up vote
                1
                down vote













                Consider a general topological space $(X, mathscrT)$ and a sequence of points $x in X^mathbbN$. One says that point $t in X$ is adherent to the sequence $x$ (some authors use the terminology 'cluster-point', but I don't fancy it so much) if:



                $$(forall V, n)(V in mathscrV_mathscrT(x) wedge n in mathbbN implies (exists m)(m geqslant n wedge x_m in V ))$$



                where $mathscrV_mathscrT(x)$ symbolizes the filter of neighbourhoods of point $x$ induced by the topology $mathscrT$. In a more descriptive fashion, $t$ is adherent to sequence $x$ if any neighbourhood of $t$ contains terms of arbitrarily high rank from the sequence $x$. If the filter of neighbourhoods of $t$ admits a countable base, then $t$ can be expressed as the limit of a subsequence of $x$. Therefore, in a space satisfying the First Axiom of Countability (i.e. all points have a countable base of neighbourhoods), the points adherent to a given sequence $x$ can be equivalently characterised as limits of subsequences of $x$.



                Now, if the space $X$ is non-empty and separable, let us fix a certain dense subset $T subseteq X$. As $X$ is non-empty, so must $T$ be. It is not difficult to show that a non-empty countable set $M$ admits a surjection $sigma: mathbbN rightarrow M$ such that for each $t in M$ the fibre $sigma^-1(t)$ be infinite.



                Consider such a surjection $sigma: mathbbN rightarrow T$ and define the sequence $t=(sigma(n))_n in mathbbN$ (which is actually the graphic of map $sigma$). The condition on the cardinality of the fibers ensures that any element $x in T$ is the limit of a (constant) subsequence of $t$. If we furthermore assume that the space $(X, mathscrT)$ is $T_1$, then any $x in X setminus T$ will be an accumulation point of $T$ and thus necessarily adherent to sequence $t$.



                To conclude, given a topological space $(X, mathscrT)$ that is non-empty, separable, first countable and $T_1$, one can always find a sequence $t$ of points within the space such that each element $x in X$ be expressible as the limit of a subsequence of $t$. This applies in particular to the topological space $(mathbbR, mathscrO)$, where $mathscrO$ denotes the (usual) order topology.






                share|cite|improve this answer






















                  up vote
                  1
                  down vote










                  up vote
                  1
                  down vote









                  Consider a general topological space $(X, mathscrT)$ and a sequence of points $x in X^mathbbN$. One says that point $t in X$ is adherent to the sequence $x$ (some authors use the terminology 'cluster-point', but I don't fancy it so much) if:



                  $$(forall V, n)(V in mathscrV_mathscrT(x) wedge n in mathbbN implies (exists m)(m geqslant n wedge x_m in V ))$$



                  where $mathscrV_mathscrT(x)$ symbolizes the filter of neighbourhoods of point $x$ induced by the topology $mathscrT$. In a more descriptive fashion, $t$ is adherent to sequence $x$ if any neighbourhood of $t$ contains terms of arbitrarily high rank from the sequence $x$. If the filter of neighbourhoods of $t$ admits a countable base, then $t$ can be expressed as the limit of a subsequence of $x$. Therefore, in a space satisfying the First Axiom of Countability (i.e. all points have a countable base of neighbourhoods), the points adherent to a given sequence $x$ can be equivalently characterised as limits of subsequences of $x$.



                  Now, if the space $X$ is non-empty and separable, let us fix a certain dense subset $T subseteq X$. As $X$ is non-empty, so must $T$ be. It is not difficult to show that a non-empty countable set $M$ admits a surjection $sigma: mathbbN rightarrow M$ such that for each $t in M$ the fibre $sigma^-1(t)$ be infinite.



                  Consider such a surjection $sigma: mathbbN rightarrow T$ and define the sequence $t=(sigma(n))_n in mathbbN$ (which is actually the graphic of map $sigma$). The condition on the cardinality of the fibers ensures that any element $x in T$ is the limit of a (constant) subsequence of $t$. If we furthermore assume that the space $(X, mathscrT)$ is $T_1$, then any $x in X setminus T$ will be an accumulation point of $T$ and thus necessarily adherent to sequence $t$.



                  To conclude, given a topological space $(X, mathscrT)$ that is non-empty, separable, first countable and $T_1$, one can always find a sequence $t$ of points within the space such that each element $x in X$ be expressible as the limit of a subsequence of $t$. This applies in particular to the topological space $(mathbbR, mathscrO)$, where $mathscrO$ denotes the (usual) order topology.






                  share|cite|improve this answer












                  Consider a general topological space $(X, mathscrT)$ and a sequence of points $x in X^mathbbN$. One says that point $t in X$ is adherent to the sequence $x$ (some authors use the terminology 'cluster-point', but I don't fancy it so much) if:



                  $$(forall V, n)(V in mathscrV_mathscrT(x) wedge n in mathbbN implies (exists m)(m geqslant n wedge x_m in V ))$$



                  where $mathscrV_mathscrT(x)$ symbolizes the filter of neighbourhoods of point $x$ induced by the topology $mathscrT$. In a more descriptive fashion, $t$ is adherent to sequence $x$ if any neighbourhood of $t$ contains terms of arbitrarily high rank from the sequence $x$. If the filter of neighbourhoods of $t$ admits a countable base, then $t$ can be expressed as the limit of a subsequence of $x$. Therefore, in a space satisfying the First Axiom of Countability (i.e. all points have a countable base of neighbourhoods), the points adherent to a given sequence $x$ can be equivalently characterised as limits of subsequences of $x$.



                  Now, if the space $X$ is non-empty and separable, let us fix a certain dense subset $T subseteq X$. As $X$ is non-empty, so must $T$ be. It is not difficult to show that a non-empty countable set $M$ admits a surjection $sigma: mathbbN rightarrow M$ such that for each $t in M$ the fibre $sigma^-1(t)$ be infinite.



                  Consider such a surjection $sigma: mathbbN rightarrow T$ and define the sequence $t=(sigma(n))_n in mathbbN$ (which is actually the graphic of map $sigma$). The condition on the cardinality of the fibers ensures that any element $x in T$ is the limit of a (constant) subsequence of $t$. If we furthermore assume that the space $(X, mathscrT)$ is $T_1$, then any $x in X setminus T$ will be an accumulation point of $T$ and thus necessarily adherent to sequence $t$.



                  To conclude, given a topological space $(X, mathscrT)$ that is non-empty, separable, first countable and $T_1$, one can always find a sequence $t$ of points within the space such that each element $x in X$ be expressible as the limit of a subsequence of $t$. This applies in particular to the topological space $(mathbbR, mathscrO)$, where $mathscrO$ denotes the (usual) order topology.







                  share|cite|improve this answer












                  share|cite|improve this answer



                  share|cite|improve this answer










                  answered Dec 9 at 14:09









                  ΑΘΩ

                  2363




                  2363












                      Popular posts from this blog

                      How to check contact read email or not when send email to Individual?

                      Bahrain

                      Postfix configuration issue with fips on centos 7; mailgun relay